Exterior and limit points: Quick Question

  • Thread starter Thread starter Buri
  • Start date Start date
  • Tags Tags
    Limit Points
Click For Summary
SUMMARY

The assertion that X\lim(A) equals ext(A) for a subset A of a metric space X is false. The exterior of A is defined as X minus the closure of A, which does not equate to the set of limit points of A. The discussion highlights the importance of considering isolated points, as demonstrated with the example A = [0, 1] ∪ {2} in the real numbers R, where 2 is an isolated point that affects the relationship between limit points and the exterior.

PREREQUISITES
  • Understanding of metric spaces
  • Familiarity with the concepts of limit points and closure
  • Knowledge of isolated points in topology
  • Basic proficiency in set theory
NEXT STEPS
  • Study the definitions and properties of limit points in metric spaces
  • Explore the concept of closure and its relationship to exterior points
  • Investigate examples of isolated points and their implications in topology
  • Learn about the topology of the real numbers, particularly in relation to closed and open sets
USEFUL FOR

Students of mathematics, particularly those studying topology, as well as educators and anyone seeking to deepen their understanding of metric spaces and the properties of limit points.

Buri
Messages
271
Reaction score
0
Is the following true?

X\lim(A) = ext(A), where A is a subset of a metric space X.

I think I've found a proof, but I don't feel very secure about it. So could someone just tell me if this is a correct assertion? What I'm trying to prove is that lim(A) is closed.

Thanks
 
Physics news on Phys.org
No, this is false. The exterior of A can be expressed as X minus something, but that "something" is not the set of limit points of A. (As a hint, think about isolated points.)
 
Ahh really? Hmm I'm going to ahve to think about it more then...damnit
 
In R, consider A= [0, 1]\cup \{2\}.
 
Yup, those damn isolated points. Thanks!
 
Question: A clock's minute hand has length 4 and its hour hand has length 3. What is the distance between the tips at the moment when it is increasing most rapidly?(Putnam Exam Question) Answer: Making assumption that both the hands moves at constant angular velocities, the answer is ## \sqrt{7} .## But don't you think this assumption is somewhat doubtful and wrong?

Similar threads

  • · Replies 5 ·
Replies
5
Views
2K
Replies
32
Views
3K
  • · Replies 16 ·
Replies
16
Views
3K
Replies
2
Views
2K
  • · Replies 18 ·
Replies
18
Views
3K
  • · Replies 3 ·
Replies
3
Views
1K
  • · Replies 8 ·
Replies
8
Views
3K
  • · Replies 5 ·
Replies
5
Views
1K
Replies
2
Views
2K
  • · Replies 2 ·
Replies
2
Views
2K